1

When we calculate the relativistic angular momentum of a particle in the direction of the $z$-axis, what relativistic mass should we use? My hypothesis is that the relativistic mass used for such a calculation does not depend on the particle’s speed in the $z$-direction; rather the relativistic mass used for such a calculation should depend on the speed of the particle on the $xy$-plane, i.e., the Lorentz factor should only use the square of the speed on the $xy$-plane. Is my hypothesis correct?

Relativistic angular momentum in a related discussion suggests that the component of the relativistic angular momentum along the direction of motion between two inertial frames is the same for both frames. It can be inferred from this that the relativistic mass used for calculating the relativistic angular momentum in that direction is independent of the momentum in that direction. Your advice will be much appreciated.

Qmechanic
  • 201,751
Damon
  • 197
  • 8
    Just don't use relativistic mass. It makes everything a mess, and that's why relativists stopped using it 50 years ago. See https://physics.stackexchange.com/questions/133376/why-is-there-a-controversy-on-whether-mass-increases-with-speed . –  Dec 18 '19 at 01:01
  • Is your book using relativistic mass? – G. Smith Dec 18 '19 at 01:12

3 Answers3

3

enter image description here

Consider a particle $\,\rm P\,$ of rest mass $\,m_0\,$ moving in an inertial system $\,\mathrm S\boldsymbol{\equiv}\mathrm Ox_1x_2x_3t\,$ with velocity
\begin{equation} \mathbf{u}\boldsymbol{=}\dfrac{\mathrm d\mathbf{x}}{\mathrm dt}\boldsymbol{=} \begin{bmatrix} u_1\\ u_2\\ u_3 \end{bmatrix} \tag{01}\label{01} \end{equation} For the relativistic linear and angular momentum 3-vectors $\,\mathbf{p}\,$ and $\,\mathbf{h}\,$ respectively we have \begin{equation} \mathbf{p}\boldsymbol{=}\gamma_{\rm u}m_0\mathbf{u}\boldsymbol{=} \begin{bmatrix} \gamma_{\rm u}m_0u_1\\ \gamma_{\rm u}m_0u_2\\ \gamma_{\rm u}m_0u_3 \end{bmatrix} \boldsymbol{=} \begin{bmatrix} p_1\\ p_2\\ p_3 \end{bmatrix} \quad \text{where} \quad \gamma_{\rm u} \boldsymbol{=} \left(1\boldsymbol{-}\dfrac{u^2}{c^2}\right)^{\boldsymbol{-}\frac12} \tag{02}\label{02} \end{equation} and \begin{equation} \mathbf{h}\boldsymbol{=}\mathbf{x}\boldsymbol{\times}\mathbf{p}\boldsymbol{=} \begin{bmatrix} x_2p_3\boldsymbol{-}x_3p_2\\ x_3p_1\boldsymbol{-}x_1p_3\\ x_1p_2\boldsymbol{-}x_2p_1 \end{bmatrix} \boldsymbol{=} \begin{bmatrix} \gamma_{\rm u}m_0\left(x_2u_3\boldsymbol{-}x_3u_2\right)\\ \gamma_{\rm u}m_0\left(x_3u_1\boldsymbol{-}x_1u_3\right)\\ \gamma_{\rm u}m_0\left(x_1u_2\boldsymbol{-}x_2u_1\right) \end{bmatrix} \boldsymbol{=} \begin{bmatrix} h_1\\ h_2\\ h_3 \end{bmatrix} \tag{03}\label{03} \end{equation} In equations \eqref{02} and \eqref{03} the only mass term is the rest one $\,m_0$. There is no such a quantity like 'relativistic mass' and it would be a good practice not to use this term for $\,\gamma_{\rm u}m_0\,$ as suggested by the experts in the field (see the comments under the question).

Now, to see how the angular momentum is transformed under a Lorentz transformation let an inertial system $\,\mathrm S'\boldsymbol{\equiv}\mathrm O'x'_1x'_2x'_3t'\,$ translating with respect to the inertial system $\:\mathrm S\:$ with constant velocity \begin{align} \boldsymbol{\upsilon} & \boldsymbol{=}\left(\upsilon_{1},\upsilon_{2},\upsilon_{3}\right) \tag{04a}\label{04a}\\ \upsilon & \boldsymbol{=}\Vert \boldsymbol{\upsilon} \Vert \boldsymbol{=} \sqrt{ \upsilon^2_{1}\boldsymbol{+}\upsilon^2_{2}\boldsymbol{+}\upsilon^2_{3}}\:\in \left(0,c\right) \tag{04b}\label{04b} \end{align} as in Figure-01.

The Lorentz transformation is \begin{align} \mathbf{x}^{\boldsymbol{\prime}} & \boldsymbol{=} \mathbf{x}\boldsymbol{+} \dfrac{\gamma_\upsilon^2}{c^2 \left(\gamma_\upsilon\boldsymbol{+}1\right)}\left(\boldsymbol{\upsilon}\boldsymbol{\cdot} \mathbf{x}\right)\boldsymbol{\upsilon}\boldsymbol{-}\gamma_\upsilon\boldsymbol{\upsilon}\,t \tag{05a}\label{05a}\\ t^{\boldsymbol{\prime}} & \boldsymbol{=} \gamma_\upsilon\left(t\boldsymbol{-} \dfrac{\boldsymbol{\upsilon}\boldsymbol{\cdot} \mathbf{x}}{c^2}\right) \tag{05b}\label{05b}\\ \gamma_\upsilon & \boldsymbol{=} \left(1\boldsymbol{-}\dfrac{\upsilon^2}{c^2}\right)^{\boldsymbol{-}\frac12} \tag{05c}\label{05c} \end{align}

For the Lorentz transformation \eqref{05a}-\eqref{05c} it could be proved that the pair of the angular momentum 3-vector $\,\mathbf{h}\,$ and the following defined 3-vector \begin{equation} \boldsymbol{\varrho}\boldsymbol{=}\gamma_{\rm u}m_0\mathbf{x}\boldsymbol{-}\mathbf{p}\,t\boldsymbol{=}\gamma_{\rm u}m_0\left(\mathbf{x}\boldsymbol{-}\mathbf{u}\,t\right) \boldsymbol{=} \begin{bmatrix} \gamma_{\rm u}m_0\left(x_1\boldsymbol{-}u_1\,t\right)\\ \gamma_{\rm u}m_0\left(x_2\boldsymbol{-}u_2\,t\right)\\ \gamma_{\rm u}m_0\left(x_3\boldsymbol{-}u_3\,t\right) \end{bmatrix} \boldsymbol{=} \begin{bmatrix} \varrho_1\\ \varrho_2\\ \varrho_3 \end{bmatrix} \tag{06}\label{06} \end{equation} is transformed as follows \begin{align} \mathbf{h}' & \boldsymbol{=}\gamma_\upsilon \mathbf{h}\,\boldsymbol{-}\,\dfrac{\gamma_\upsilon^2}{c^2 \left(\gamma_\upsilon\boldsymbol{+}1\right)}\left(\mathbf{h}\boldsymbol{\cdot} \boldsymbol{\upsilon}\right)\boldsymbol{\upsilon}\,\boldsymbol{+}\,\gamma_\upsilon\left(\boldsymbol{\upsilon}\boldsymbol{\times}\boldsymbol{\varrho}\right) \tag{07a}\label{07a}\\ \boldsymbol{\varrho}' & \boldsymbol{=} \gamma_\upsilon \boldsymbol{\varrho}\,\boldsymbol{-}\,\dfrac{\gamma_\upsilon^2}{c^2 \left(\gamma_\upsilon\boldsymbol{+}1\right)}\left(\boldsymbol{\varrho}\boldsymbol{\cdot} \boldsymbol{\upsilon}\right)\boldsymbol{\upsilon}\,\boldsymbol{-}\,\dfrac{\gamma_\upsilon}{c^2}\left(\boldsymbol{\upsilon}\boldsymbol{\times}\mathbf{h}\right) \tag{07b}\label{07b} \end{align}

Now, for the component of the relativistic angular momentum along the direction of motion \begin{equation} \mathbf{n} \boldsymbol{=}\dfrac{\boldsymbol{\upsilon}}{\Vert \boldsymbol{\upsilon} \Vert}\boldsymbol{=}\dfrac{\boldsymbol{\upsilon}}{ \upsilon} \tag{08}\label{08} \end{equation} we have from equation \eqref{07a} \begin{align} \left(\mathbf{h}'\boldsymbol{\cdot}\mathbf{n}\right)\mathbf{n} & \boldsymbol{=}\dfrac{\left(\mathbf{h}'\boldsymbol{\cdot}\boldsymbol{\upsilon}\right)\boldsymbol{\upsilon}}{\upsilon^2}\boldsymbol{=}\gamma_\upsilon\dfrac{\left(\mathbf{h}\boldsymbol{\cdot}\boldsymbol{\upsilon}\right)\boldsymbol{\upsilon}}{\upsilon^2}\,\boldsymbol{-}\,\dfrac{\upsilon^2\gamma_\upsilon^2}{c^2 \left(\gamma_\upsilon\boldsymbol{+}1\right)}\dfrac{\left(\mathbf{h}\boldsymbol{\cdot}\boldsymbol{\upsilon}\right)\boldsymbol{\upsilon}}{\upsilon^2} \nonumber\\ &\boldsymbol{=}\underbrace{\left[\gamma_\upsilon\,\boldsymbol{-}\,\dfrac{\upsilon^2\gamma_\upsilon^2}{c^2 \left(\gamma_\upsilon\boldsymbol{+}1\right)}\right]}_{\boldsymbol{=}1}\dfrac{\left(\mathbf{h}\boldsymbol{\cdot}\boldsymbol{\upsilon}\right)\boldsymbol{\upsilon}}{\upsilon^2}\boldsymbol{=}\left(\mathbf{h}\boldsymbol{\cdot}\mathbf{n}\right)\mathbf{n} \tag{09}\label{09} \end{align} that is \begin{equation} \left(\mathbf{h}'\boldsymbol{\cdot}\mathbf{n}\right)\mathbf{n} \boldsymbol{=}\left(\mathbf{h}\boldsymbol{\cdot}\mathbf{n}\right)\mathbf{n} \tag{10}\label{10} \end{equation} So the component of the relativistic angular momentum along the direction of motion between these two inertial frames is the same in both frames. This result has nothing to do with any concept of 'relativistic mass'.

$\boldsymbol{=\!=\!=\!==\!=\!=\!==\!=\!=\!==\!=\!=\!==\!=\!=\!==\!=\!=\!==\!=\!=\!==\!=\!=\!==\!=\!=\!==\!=\!=\!==\!=\!=\!==\!=\!=\!=}$

$\textbf{ADDENDUM 01}$

Note that for the Lorentz transformation \eqref{05a}-\eqref{05c} the vectors $\:\mathbf{E}\:$ and $\:\mathbf{B}\:$ of the electromagnetic field are transformed as follows \begin{align} \mathbf{E}' & \boldsymbol{=}\gamma_\upsilon \mathbf{E}\,\boldsymbol{-}\,\dfrac{\gamma_\upsilon^2}{c^2 \left(\gamma_\upsilon\boldsymbol{+}1\right)}\left(\mathbf{E}\boldsymbol{\cdot} \boldsymbol{\upsilon}\right)\boldsymbol{\upsilon}\,\boldsymbol{+}\,\gamma_\upsilon\left(\boldsymbol{\upsilon}\boldsymbol{\times}\mathbf{B}\right) \tag{11a}\label{11a}\\ \mathbf{B}' & \boldsymbol{=} \gamma_\upsilon \mathbf{B}\,\boldsymbol{-}\,\dfrac{\gamma_\upsilon^2}{c^2 \left(\gamma_\upsilon\boldsymbol{+}1\right)}\left(\mathbf{B}\boldsymbol{\cdot} \boldsymbol{\upsilon}\right)\boldsymbol{\upsilon}\,\boldsymbol{-}\,\dfrac{\gamma_\upsilon}{c^2}\left(\boldsymbol{\upsilon}\boldsymbol{\times}\mathbf{E}\right) \tag{11b}\label{11b} \end{align} Comparing equations \eqref{07a},\eqref{07b} with \eqref{11a},\eqref{11b} we conclude that the pair of 3-vectors $\:\left(\mathbf{h},\boldsymbol{\varrho}\right)\:$ is transformed as the pair of 3-vectors $\:\left(\mathbf{E},\mathbf{B}\right)$. But as for the electromagnetic field we construct from $\,\left(\mathbf{E},\mathbf{B}\right)\,$ the anti-symmetric four-tensor \begin{equation} \mathrm F^{\mu\nu}\boldsymbol{=} \begin{bmatrix} 0 & \boldsymbol{-}E_{1} & \boldsymbol{-}E_{2} & \boldsymbol{-}E_{3} \\ E_{1} & \hphantom{\boldsymbol{-}} 0 \hphantom{_{1}} & \boldsymbol{-}cB_{3} & \hphantom{\boldsymbol{-}}cB_{2} \\ E_{2} & \hphantom{\boldsymbol{-}}cB_{3} & \hphantom{\boldsymbol{-}} 0 & \boldsymbol{-}cB_{1} \\ E_{3} & \boldsymbol{-}cB_{2} & \hphantom{\boldsymbol{-}}cB_{1} & \hphantom{\boldsymbol{-}} 0 \end{bmatrix} \tag{12}\label{12} \end{equation} so from the pair of 3-vectors $\:\left(\mathbf{h},\boldsymbol{\varrho}\right)\:$ we construct the anti-symmetric four-tensor \begin{equation} \mathrm H^{\mu\nu}\boldsymbol{=} \begin{bmatrix} 0 & \boldsymbol{-}h_{1} & \boldsymbol{-}h_{2} & \boldsymbol{-}h_{3} \\ h_{1} & \hphantom{\boldsymbol{-}} 0 \hphantom{_{1}} & \boldsymbol{-}c\varrho_{3} & \hphantom{\boldsymbol{-}}c\varrho_{2} \\ h_{2} & \hphantom{\boldsymbol{-}}c\varrho_{3} & \hphantom{\boldsymbol{-}} 0 & \boldsymbol{-}c\varrho_{1} \\ h_{3} & \boldsymbol{-}c\varrho_{2} & \hphantom{\boldsymbol{-}}c\varrho_{1} & \hphantom{\boldsymbol{-}} 0 \end{bmatrix} \tag{13}\label{13} \end{equation} which represents the relativistic angular momentum of a particle.

$\boldsymbol{=\!=\!=\!==\!=\!=\!==\!=\!=\!==\!=\!=\!==\!=\!=\!==\!=\!=\!==\!=\!=\!==\!=\!=\!==\!=\!=\!==\!=\!=\!==\!=\!=\!==\!=\!=\!=}$

$\textbf{ADDENDUM 02 : The Lorentz transformation of velocity 3-vectors}$

This $\textbf{ADDENDUM 02}$ is motivated by the following OP's comment with incorrect arguments :

Suppose frame $\,\mathrm S'\,$ moves relative to frame $\,\mathrm S\,$ in the $\,x_3\,$ direction with speed $\,u_3\,$ so that the particle's velocity relative to $\,\mathrm S'\,$ in the $\,x_3\,$ direction, $\,u'_3\,$, is 0; and $u'_1=u_1,u'_2=u_2$. The Lorentz factor for the particle's momentum relative to frame $\,\mathrm S\,$ will be $\gamma_{u'} = \frac {1}{\sqrt {1- \frac {u_1^2+u_2^2}{c^2} } } \neq \gamma_{u}$. However, $h_3=\gamma_{u} m_0\left(x_1u_2\boldsymbol{-}x_2u_1\right)$ but $h'_3=\gamma_{u'} m_0\left(x_1u_2\boldsymbol{-}x_2u_1\right) \neq h_3$. But we had concluded that $h'_3= h_3$. Hence, we have a contradiction. Something is wrong.

The differential version of the Lorentz transformation \eqref{05a}-\eqref{05c} is \begin{align} \mathrm d\mathbf{x}^{\boldsymbol{\prime}} & \boldsymbol{=} \mathrm d\mathbf{x}\boldsymbol{+} \dfrac{\gamma_\upsilon^2}{c^2 \left(\gamma_\upsilon\boldsymbol{+}1\right)}\left(\boldsymbol{\upsilon}\boldsymbol{\cdot} \mathrm d\mathbf{x}\right)\boldsymbol{\upsilon}\boldsymbol{-}\gamma_\upsilon\boldsymbol{\upsilon}\,\mathrm dt \tag{14a}\label{14a}\\ \mathrm dt^{\boldsymbol{\prime}} & \boldsymbol{=} \gamma_\upsilon\left(\mathrm dt\boldsymbol{-} \dfrac{\boldsymbol{\upsilon}\boldsymbol{\cdot} \mathrm d\mathbf{x}}{c^2}\right) \tag{14b}\label{14b}\\ \gamma_\upsilon & \boldsymbol{=} \left(1\boldsymbol{-}\dfrac{\upsilon^2}{c^2}\right)^{\boldsymbol{-}\frac12} \tag{14c}\label{14c} \end{align} Now, suppose that a particle is moving relative to frame $\,\mathrm S\,$ with velocity \begin{equation} \mathbf{u}\boldsymbol{=}\dfrac{\mathrm d\mathbf{x}}{\mathrm dt} \tag{15}\label{15} \end{equation} To find the velocity of the particle relative to the frame $\,\mathrm S'\,$ \begin{equation} \mathbf{u'}\boldsymbol{=}\dfrac{\mathrm d\mathbf{x'}}{\mathrm dt'} \tag{16}\label{16} \end{equation} we divide equations \eqref{14a} and \eqref{14b} side by side and we have \begin{equation} \left(\dfrac{\mathrm d\mathbf{x'}}{\mathrm dt'}\right)\boldsymbol{=}\dfrac{\left(\dfrac{\mathrm d\mathbf{x}}{\mathrm dt}\right)\boldsymbol{+} \dfrac{\gamma_\upsilon^2}{c^2 \left(\gamma_\upsilon\boldsymbol{+}1\right)}\left[\boldsymbol{\upsilon}\boldsymbol{\cdot} \left(\dfrac{\mathrm d\mathbf{x}}{\mathrm dt}\right)\right]\boldsymbol{\upsilon}\boldsymbol{-}\gamma_\upsilon\boldsymbol{\upsilon}}{\gamma_\upsilon\left[1\boldsymbol{-} \dfrac{\boldsymbol{\upsilon}\boldsymbol{\cdot} \left(\dfrac{\mathrm d\mathbf{x}}{\mathrm dt}\right)}{c^2}\right]} \tag{17}\label{17} \end{equation} that is \begin{equation} \mathbf{u'}\boldsymbol{=}\dfrac{\mathbf{u}\boldsymbol{+} \dfrac{\gamma_\upsilon^2}{c^2 \left(\gamma_\upsilon\boldsymbol{+}1\right)}\left(\boldsymbol{\upsilon}\boldsymbol{\cdot} \mathbf{u}\right)\boldsymbol{\upsilon}\boldsymbol{-}\gamma_\upsilon\boldsymbol{\upsilon}}{\gamma_\upsilon\left(1\boldsymbol{-} \dfrac{\boldsymbol{\upsilon}\boldsymbol{\cdot}\mathbf{u}}{c^2}\right)} \tag{18}\label{18} \end{equation} So, for the special case of motions of systems $\,\mathrm S,\mathrm S'\,$ and particle, as in OP's comment, we have \begin{equation} \mathbf{u}\boldsymbol{=} \begin{bmatrix} u_1\\ u_2\\ u_3 \end{bmatrix}\,, \quad \boldsymbol{\upsilon}\boldsymbol{=} \begin{bmatrix} 0\\ 0\\ u_3 \end{bmatrix} \,, \quad u_3\boldsymbol{\ne}0 \tag{19}\label{19} \end{equation} From equation \eqref{18} \begin{equation} \mathbf{u'}\boldsymbol{=} \begin{bmatrix} u'_1\\ u'_2\\ u'_3 \end{bmatrix}\boldsymbol{=}\gamma_\upsilon \begin{bmatrix} u_1\\ u_2\\ 0 \end{bmatrix} \quad =\!=\!=\!\Longrightarrow \quad \ \begin{bmatrix} u'_1\boldsymbol{=}\gamma_\upsilon u_1\boldsymbol{=}\left(1\boldsymbol{-}\dfrac{u^2_3}{c^2}\right)^{\boldsymbol{-}\frac12}u_1\\ u'_2\boldsymbol{=}\gamma_\upsilon u_2\boldsymbol{=}\left(1\boldsymbol{-}\dfrac{u^2_3}{c^2}\right)^{\boldsymbol{-}\frac12}u_2\\ u'_3\boldsymbol{=}0\hphantom{_\upsilon u_2\boldsymbol{=}\left(1\boldsymbol{-}\dfrac{u^2_3}{c^2}\right)^{\boldsymbol{-}\frac12}u_2} \end{bmatrix} \tag{20}\label{20} \end{equation} From \eqref{20} $\,u'_1\boldsymbol{=}u_1\,$ and/or $\,u'_2\boldsymbol{=}u_2\,$ if and only if $\,u'_1\boldsymbol{=}0\boldsymbol{=}u_1\,$ and/or $\,u'_2\boldsymbol{=}0\boldsymbol{=}u_2\,$ respectively.

Frobenius
  • 15,613
  • Thank you, Frobenius, for your help. I should spell out my question more clearly as follows. A particle is moving relative to an inertial frame and in general has three components in its velocity. I wish to evaluate the component of the relativistic angular momentum in one of the three directions, e.g., the z direction. In the Lorentz factor, should the velocity squared exclude or include the contribution from the velocity in the z direction? My reasoning is that it should be excluded. Otherwise, it might contradict your conclusion of identical angular momentum w.r.t. two frames. Am I correct? – Damon Dec 20 '19 at 02:46
  • Let us say the second frame moves relative to the first frame in the x_3 direction with speed u_3 so that the velocity relative to the second frame in that direction is 0. The Lorentz factor for the second frame will be smaller than the one for the first frame because of the 0 speed in x_3 direction. But the speed in the x_1 direction is the same for both frames; similarly for the speed in the x_2 direction. Overall, the relativistic angular momenta for the two frames are different because they have different Lorentz factors while the rest of the expression is identical for both cases. Problem – Damon Dec 21 '19 at 03:00
  • Suppose frame S′ moves relative to frame S in the $x_3$ direction with speed $u_3$ so that the particle's velocity relative to S′ in the $x_3$ direction, $u'3$, is 0; and $u'_1 = u_1, u'_2=u_2$. The Lorentz factor for the particle's momentum relative to frame S′ will be $\gamma{u'} = \frac {1}{\sqrt {1- \frac {u_1^2+u_2^2}{c^2} } } \neq \gamma_{u}$. However, $h_3=\gamma_{u} m_0\left(x_1u_2\boldsymbol{-}x_2u_1\right)$ but $h'3=\gamma{u'} m_0\left(x_1u_2\boldsymbol{-}x_2u_1\right) \neq h_3$. But we had concluded that $h'_3= h_3$. Hence, we have a contradiction. Something is wrong. – Damon Dec 21 '19 at 16:49
  • You are correct. $u'2\boldsymbol{=}\gamma\upsilon u_2$ and $u'1\boldsymbol{=}\gamma\upsilon u_1$. For $h_3=\gamma_{u} m_0\left(x_1u_2\boldsymbol{-}x_2u_1\right)$ and $h'3=\gamma{u'} m_0\left(x_1u'2\boldsymbol{-}x_2u'_1\right)$ to be the same, we require $\gamma{u'} u'2 = \gamma_u u_2$ and $\gamma{u'} u'1 = \gamma_u u_1$. This in turn requires $\gamma{u'} \gamma_\upsilon = \gamma_u$. But I have worked out $\gamma_{u'} \gamma_\upsilon$ which is not quite $\gamma_u$. I am still puzzled. I may be missing something. Thanks for your help so far. – Damon Dec 22 '19 at 01:10
  • In general $$\dfrac{\gamma_{u'}}{\gamma_{u}}\boldsymbol{=}\gamma_{\upsilon}\left(1\boldsymbol{-}\dfrac{\mathbf{u}\boldsymbol{\cdot}\boldsymbol{\upsilon}}{c^2}\right)$$ or inversely $$\dfrac{\gamma_{u}}{\gamma_{u'}}\boldsymbol{=}\gamma_{\upsilon}\left(1\boldsymbol{+}\dfrac{\mathbf{u'}\boldsymbol{\cdot}\boldsymbol{\upsilon}}{c^2}\right)$$ From the second equation and since here $\mathbf{u'}\boldsymbol{\cdot}\boldsymbol{\upsilon}\boldsymbol{=}0$ you have your equation $\gamma_{u'} \gamma_\upsilon = \gamma_u$. – Frobenius Dec 22 '19 at 01:46
  • The relation between the $,\gamma\boldsymbol{-}$factors is very difficult to be proved algebraically. The proof is based on the relations between the time coordinates $,t,t' ,$ and $,\tau,$(the proper time) in the 3 frames $,\mathrm S,$,$,\mathrm S',$ and the rest frame of the particle $,\mathrm S_0,$ respectively. For the "How to" see in my answer here : How to add together non-parallel rapidities?. – Frobenius Dec 22 '19 at 06:20
  • Prompted by your response above, I checked my algebra for an expression of $\gamma_{u'} \gamma_\upsilon$. There was an error and the corrected expression is indeed identical to $\gamma_\upsilon$. This is consistent with your proof above and in your other linked page. We can confidently conclude that the component of the relativistic angular momentum along the direction of motion between two inertial frames is invariant in both frames. That is a great relief for me as the particular case I am working on strongly suggests this. Nature is consistent with itself! I am grateful for your help. – Damon Dec 23 '19 at 14:56
  • If I wish to cite a proof for this invariant relativistic angular momentum along the direction of motion between two inertial frames, apart from citing this page and your other useful linked page, which book(s) or paper(s) would you recommend for citing for support? – Damon Dec 23 '19 at 15:06
  • 'Introduction to Special Relativity' [Rindler] – Frobenius Dec 23 '19 at 16:17
-1

A particle is sitting still in an Ecuadorian lab. It has orbital angular momentum because the earth is spinning. Also in this lab there is a linear particle accelerator, it runs from north to south.

When said particle is accelerated in said accelerator, first the E-field in the accelerator has orbital angular momentum, because the earth is spinning, after the particle has been accelerated to high speed, the E-field has lost some orbital angular momentum, and the particle has gained some orbital angular momentum.

Here's the mathematical part:

The orbital angular momentum of said particle is proportional to its west-east momentum, which is proportional to its relativistic mass. Or it's proportional to the gamma factor, if that's better.

stuffu
  • 1,978
  • 11
  • 11
-2

Use relativistic mass.

Remember that adding non-spinning mass to spinning object slows down the spinning of the object.

I'll give some examples. They all involve loss of mass and increase of spinning rate, instead of gain of mass and decrease of spinning rate. It's just easier to come up with the former kind of examples.

For example if a we have a spinning water tank that leaks water out of its pole, when that tank has lost half of its mass, then its spinning rate has doubled.

If a truck moving at speed 0.87c carries a flywheel, stopping the truck causes the flywheel lose half of its relativistic mass and its spinning rate doubles.

If a very hot spinning wheel radiates half of its mass away, its spinning rate stays the same. If we paint that wheel so that it radiates only from its poles, then its spinning rate doubles when its total mass halves. (In the latter case the radiation has no angular momentum)

The reader is encouraged to ponder the similarities between the flywheel in the braking truck and the spinning wheel cooling in space. (One thing that is same in both cases is that mass decreases while angular momentum stays the same)

Here is an example for people who dislike relativistic mass:

There is a fleet of two trucks, both of which carry a spinning flywheel. One truck starts accelerating towards south, other one towards north. The part of this system called "fuel tanks" loses rest mass. All other parts of the fleet gain rest mass, including the part named "two flywheels". The angular momentum of "two flywheels" is proportional to its rest mass and its angular velocity, the angular momentum of "two flywheels" stays constant, its rest mass increases, so its angular velocity must decrease.

stuffu
  • 1,978
  • 11
  • 11